LSAT and Law School Admissions Forum

Get expert LSAT preparation and law school admissions advice from PowerScore Test Preparation.

 Administrator
PowerScore Staff
  • PowerScore Staff
  • Posts: 8916
  • Joined: Feb 02, 2011
|
#35110
Complete Question Explanation

Strengthen—PR. The correct answer choice is (C)

Your task in this Strengthen question is to select the answer choice containing a principle that, once
added to the stimulus, will support the conclusion that the television station’s coverage was good
journalism. Reordered, the argument proceeds:

..... Premise: ..... the information reported was accurate

..... Premise: ..... the newscast had significantly more viewers than it normally does, because
..... ..... ..... ..... many people are curious about the politician’s nephew’s problems

..... Conclusion: ..... thus, the coverage was good journalism

Your prephrase is that the correct answer will provide a rule that will connect with the editorial’s
description of the coverage to support the conclusion that the coverage was good journalism. The
editorial provides two reasons to think the coverage was good journalism: 1) the information was
accurate; and 2) people are curious about the nephew’s problems. The correct answer likely will
provide a rule stating that coverage bearing one or both of these characteristics is good journalism.

The incorrect answers will not provide a rule connecting these characteristics of the coverage to
good journalism, or could state that these factors are related to bad journalism.

Answer choice (A): While the stimulus began with the fact that the station had been criticized,
the editorial’s conclusion did not pertain to when journalism deserves to be criticized. Instead, the
argument had to do only with what comprises good journalism.

Answer choice (B): This choice has no effect on the conclusion, because there was no fact presented
indicating intentional misrepresentation of the facts of the case.

Answer choice (C): This is the correct answer choice. This choice is correct, because it provides a
conditional rule stating that when journalism has the two factors associated with the coverage of the
politician’s nephew, then it is good journalism. The relationship could be diagrammed:

..... provides accurate information on a subject of considerable interest :arrow: good journalism

Answer choice (D): This choice is incorrect because it states something that is required if you have
good journalism, but does not provide a rule to help determine whether the characteristics of this
station’s coverage are also consistent with good journalism.

Answer choice (E): This choice provides the mistaken negation of the information helpful to
determining whether the station’s journalism was good journalism. This relationship could be
diagrammed:

..... satisfies public interest

..... ..... + ..... ..... ..... ..... :arrow: ..... good journalism

..... provides accurate information

Get the most out of your LSAT Prep Plus subscription.

Analyze and track your performance with our Testing and Analytics Package.